[obm-l] Re: [obm-l] Questão de probabilidade

2022-06-29 Por tôpico Rogerio Ponce
Ola' Vanderlei e pessoal da lista! Pediram-me para resolver o problema por inteiro. Ok, vamos la'! Em um pet shop ha' 3 gatos e 5 caes. Sabemos que 3 desses animais sao pretos, 4 sao brancos e 1 e' malhado. Alem disso, pelo menos 1 cachorro e' preto. Assinale o que for correto. 01) A probabilidad

[obm-l] Re: [obm-l] Re: [obm-l] Re: [obm-l] Re: [obm-l] Re: [obm-l] Questão de probabilidade

2022-06-24 Por tôpico Rogerio Ponce
Otima explicacao! Obrigado, Ralph! PS: e sim, a provocacao foi pra voce mesmo! :) []'s Rogerio Ponce On Wed, Jun 22, 2022 at 1:00 PM Ralph Costa Teixeira wrote: > > Ponce está provocando a gente... senti que esta flecha tinha um bocado a > minha direção... :D :D :D > > Olha, tem duas "visões

[obm-l] Re: [obm-l] Re: [obm-l] Re: [obm-l] Re: [obm-l] Questão de probabilidade

2022-06-22 Por tôpico Ralph Costa Teixeira
Ponce está provocando a gente... senti que esta flecha tinha um bocado a minha direção... :D :D :D Olha, tem duas "visões" sobre o que "probabilidade" significa. A primeira vai na linha de que só podemos falar de probabilidade sobre coisas que ainda não aconteceram. Vai nessa linha: se os evento

[obm-l] Re: [obm-l] Re: [obm-l] Re: [obm-l] Questão de probabilidade

2022-06-22 Por tôpico Rogerio Ponce
Olá Pedro e pessoal da lista! Segundo a opinião do Pedro, nao faz sentido perguntar qual a probabilidade de Jose ter conseguido um 6 ao jogar o dado ontem, pois isso ja' aconteceu, e, portanto, ja' esta' definido. Sera' que e' isso mesmo? []'s Rogerio Ponce On Mon, Jun 20, 2022 at 9:45 PM Pedr

[obm-l] Re: [obm-l] Re: [obm-l] Questão de probabilidade

2022-06-20 Por tôpico Pedro José
Eu na minha humilde opinião creio que a probabilidade exista quando pode ser uma coisa ou outra. No caso já é definido o que os animais são. Então já está tudo errado. A questão seria viável se dessem esses limitantes para uma criança que pintaria os desenhos dos animais. Aí sim há probabilidade.

[obm-l] Re: [obm-l] Questão de probabilidade

2022-06-17 Por tôpico Rogerio Ponce da Silva
Ola' Vanderlei e pessoal da lista! Sem perda de generalidade, podemos imaginar que vamos fazer o seguinte: - uma pintura preta em um dos caes, escolhido aleatoriamente - uma pintura "malhada" em um dos animais, escolhido aleatoriamente entre os 7 animais nao pintados - duas pintura pretas, em d

[obm-l] Questão de probabilidade

2022-03-16 Por tôpico Professor Vanderlei Nemitz
Bom dia! Na questão a seguir, do vestibular da UEM, penso que o espaço amostral tem 105 elementos, pois um cachorro é preto (desconsideramos esse). Porém, com esse pensamento, não consigo obter o gabarito, que diz que 02 e 16 são corretas. Alguém poderia ajudar? Muito obrigado! *Em um pet shop há

[obm-l] Re: [obm-l] Re: [obm-l] Questão sobre desigualdades

2021-04-14 Por tôpico Carlos Monteiro
De onde saiu essa desigualdade? Em qua., 14 de abr. de 2021 às 20:39, Anderson Torres < torres.anderson...@gmail.com> escreveu: > Em qua., 14 de abr. de 2021 às 15:54, Carlos Monteiro > escreveu: > > > > Encontre os valores máximo e mínimo da expressão: x/(x^2+1) + y/(y^2+1) > + z/(z^2+1) , ond

[obm-l] Re: [obm-l] Questão sobre desigualdades

2021-04-14 Por tôpico Anderson Torres
Em qua., 14 de abr. de 2021 às 15:54, Carlos Monteiro escreveu: > > Encontre os valores máximo e mínimo da expressão: x/(x^2+1) + y/(y^2+1) + > z/(z^2+1) , onde x, y e z são números reais que satisfazem x+y+z = 1. > > Verifica-se que 3(12x+1)/50 >= x/(x^2+1), e assim o valor máximo é 3/10 > >

[obm-l] Questão sobre desigualdades

2021-04-14 Por tôpico Carlos Monteiro
Encontre os valores máximo e mínimo da expressão: x/(x^2+1) + y/(y^2+1) + z/(z^2+1) , onde x, y e z são números reais que satisfazem x+y+z = 1. -- Esta mensagem foi verificada pelo sistema de antiv�rus e acredita-se estar livre de perigo.

[obm-l] Re: [obm-l] Re: [obm-l] Re: [obm-l] Questão OBM - U

2020-01-23 Por tôpico Ralph Teixeira
Seja ABCD o quadrilatero convexo, e seja P o encontro das diagonais. No triangulo APB, temos AP+PB>AB. Escreva as desigualdades analogas para os triangulos BPC, CPD e DPA. Somando-as, voce vai obter que 2(AC+BD)>perimetro=8 Ou seja, o infimo tem que ser pelo menos 4. Agora, para chegar no infim

[obm-l] Re: [obm-l] Re: [obm-l] Re: [obm-l] Re: [obm-l] Re: [obm-l] Re: [obm-l] Questão OBM - U

2020-01-23 Por tôpico Esdras Muniz
É fácil ver que esse ínfimo tem que ser no mínimo 4, basta fazer desigualdade triângulos com os triângulos que têm dois vértices comuns com o quadrilátero e o terceiro sendo a interseção das diagonais. E por esse argumento do Caio, vemos que é 4 mesmo. Em qui, 23 de jan de 2020 08:59, Caio Costa

[obm-l] Re: [obm-l] Re: [obm-l] Re: [obm-l] Re: [obm-l] Re: [obm-l] Questão OBM - U

2020-01-23 Por tôpico Caio Costa
Minimiza-se a soma das diagonais ao tomar-se um losango degenerado, com uma diagonal valendo 4 e outra valendo 0. Em qui, 23 de jan de 2020 08:34, gilberto azevedo escreveu: > Pensei em minimizar √(a² + (4-a)²) > 4 - a, devido ao fato do perímetro ser 8. > No caso obtenho o mínimo sendo 2√2, qua

[obm-l] Re: [obm-l] Re: [obm-l] Re: [obm-l] Re: [obm-l] Questão OBM - U

2020-01-23 Por tôpico gilberto azevedo
Pensei em minimizar √(a² + (4-a)²) 4 - a, devido ao fato do perímetro ser 8. No caso obtenho o mínimo sendo 2√2, quando o retângulo é um quadrado de lado 2. A soma das diagonais seria no caso 4√2, e não bate com o gabarito. Em qui, 23 de jan de 2020 08:20, Bernardo Freitas Paulo da Costa < bernard

[obm-l] Re: [obm-l] Re: [obm-l] Re: [obm-l] Questão OBM - U

2020-01-23 Por tôpico Bernardo Freitas Paulo da Costa
On Thu, Jan 23, 2020 at 7:24 AM gilberto azevedo wrote: >> On Sat, Jan 11, 2020 at 11:24 AM gilberto azevedo >> wrote: >> > >> > Qual o ínfimo sobre todos os quadriláteros convexos com perímetro 8 da >> > soma dos comprimentos de suas diagonais ? > > Tentei com o retângulo e o quadrado, poré

[obm-l] Re: [obm-l] Re: [obm-l] Questão OBM - U

2020-01-23 Por tôpico gilberto azevedo
Tentei com o retângulo e o quadrado, porém não obtive a resposta... O gabarito é 4. Em sáb, 11 de jan de 2020 12:03, Bernardo Freitas Paulo da Costa < bernardo...@gmail.com> escreveu: > On Sat, Jan 11, 2020 at 11:24 AM gilberto azevedo > wrote: > > > > Qual o ínfimo sobre todos os quadriláter

[obm-l] Re: [obm-l] Questão OBM - U

2020-01-11 Por tôpico Bernardo Freitas Paulo da Costa
On Sat, Jan 11, 2020 at 11:24 AM gilberto azevedo wrote: > > Qual o ínfimo sobre todos os quadriláteros convexos com perímetro 8 da soma > dos comprimentos de suas diagonais ? Quais são os quadriláteros que você tentaria? -- Bernardo Freitas Paulo da Costa -- Esta mensagem foi verificada pelo

[obm-l] Questão OBM - U

2020-01-11 Por tôpico gilberto azevedo
Qual o ínfimo sobre todos os quadriláteros convexos com perímetro 8 da soma dos comprimentos de suas diagonais ? -- Esta mensagem foi verificada pelo sistema de antiv�rus e acredita-se estar livre de perigo.

[obm-l] Re: [obm-l] Questão sobre equações funcionais

2019-07-28 Por tôpico Esdras Muniz
Errei, satisfaz sim :) Em dom, 28 de jul de 2019 14:21, Esdras Muniz escreveu: > Mas essa função que VC achou não satisfaz a igualdade. > > Em dom, 28 de jul de 2019 01:05, Carlos Monteiro < > cacacarlosalberto1...@gmail.com> escreveu: > >> (Questão) Encontre todas as funções f : R-> R tais que

[obm-l] Re: [obm-l] Questão sobre equações funcionais

2019-07-28 Por tôpico Esdras Muniz
Mas essa função que VC achou não satisfaz a igualdade. Em dom, 28 de jul de 2019 01:05, Carlos Monteiro < cacacarlosalberto1...@gmail.com> escreveu: > (Questão) Encontre todas as funções f : R-> R tais que > f(xy - f(x)) = x.f(y) > > Minha tentativa, não sei se está correta: > I) p(x, f(x)/(x-1)

[obm-l] Questão sobre equações funcionais

2019-07-27 Por tôpico Carlos Monteiro
(Questão) Encontre todas as funções f : R-> R tais que f(xy - f(x)) = x.f(y) Minha tentativa, não sei se está correta: I) p(x, f(x)/(x-1)): f( f(x)/(x-1) ) = 0; x diferente de 1 II) Seja c um número real tal que f(c)=0 i) fazendo x=c na equação encontrada em I: *c diferente de 1* f(0)=0

[obm-l] Re: [obm-l] Re: [obm-l] Questão de probabilidade

2018-11-06 Por tôpico Vanderlei Nemitz
Bela solução, Bruno! Muito obrigado! Em ter, 6 de nov de 2018 15:38, Bruno Visnadi Seja Pa a probabilidade de ocorrência de a. Defina Pb e Pc analogamente. > a = Pa(1-Pb)(1-Pc) > b = Pb(1-Pa)(1-Pc) > c = Pc(1-Pa)(1-Pb) > p = (1-Pa)(1-Pb)(1-Pc) > Queremos achar a razão Pa/Pc > Da equação (a - 2b)p

[obm-l] Re: [obm-l] Questão de probabilidade

2018-11-06 Por tôpico Bruno Visnadi
Seja Pa a probabilidade de ocorrência de a. Defina Pb e Pc analogamente. a = Pa(1-Pb)(1-Pc) b = Pb(1-Pa)(1-Pc) c = Pc(1-Pa)(1-Pb) p = (1-Pa)(1-Pb)(1-Pc) Queremos achar a razão Pa/Pc Da equação (a - 2b)p = ab, obtemos: (1-Pa)(1-Pb)(1-Pc)²(Pa(1-Pb) - 2Pb(1-Pa)) = PaPb(1-Pa)(1-Pb)(1-Pc)² Pa(1-Pb) - 2P

[obm-l] Questão de probabilidade

2018-11-06 Por tôpico Vanderlei Nemitz
Pessoal, alguém tem um ideia de como resolver a seguinte questão? Já tentei muita coisa, sem sucesso. Muito obrigado! Vanderlei Sejam três eventos independentes A, B e C. A probabilidade de que ocorra apenas o evento A é a, apenas o evento B é b e apenas o evento C é c. Seja p a probabilidade de

[obm-l] Re: [obm-l] Re: [obm-l] Questão do ITA

2018-10-11 Por tôpico Vanderlei Nemitz
Valeu, Ralph! Como sempre, uma explicação clara e simples! Em qua, 10 de out de 2018 17:05, Ralph Teixeira escreveu: > Note que x=5 é um possível valor que resolve aquela equação (mas, > sinceramente, não interessa, eu faria o raciocínio abaixo com qualquer > número). > > Então qualquer polinômi

[obm-l] Re: [obm-l] Questão do ITA

2018-10-10 Por tôpico Ralph Teixeira
Note que x=5 é um possível valor que resolve aquela equação (mas, sinceramente, não interessa, eu faria o raciocínio abaixo com qualquer número). Então qualquer polinômio que satisfaça f(1)=5, f(-1)=10 e f(0)=20 automaticamente satisfaz todas as condições do enunciado (note que a_0=f(0)). Em outra

[obm-l] Questão do ITA

2018-10-10 Por tôpico Vanderlei Nemitz
Bom dia, pessoal! Encontrei essa questão, que diz ser do ITA (eu particularmente não encontrei na internet). Como a resposta é E (nenhuma das anteriores), não sei se é possível provar que as anteriores são falsas. Eu não consegui concluir coisa alguma. *Seja f(x) = am.x^m + am–1.x^(m–1) + ... + a1

[obm-l] Re: [obm-l] Questão do Gandhi

2018-08-15 Por tôpico gilberto azevedo
Só uma ressalva, alí depois do "ou a+1 será par, e a ... " Não tem esse "a" no final, erro de digitação. Em Qua, 15 de ago de 2018 18:02, gilberto azevedo escreveu: > Supondo que b>a, então b = a+1 > Logo : > D = a² + (a+1)² + (a*(a+1))² > D = a² + a² + 2a + 1 + (a²+a)² > D = 2a² + 2a + 1 + (a²+

[obm-l] Re: [obm-l] Questão do Gandhi

2018-08-15 Por tôpico Claudio Buffara
D = a^2 + (a+1)^2 + a^2*(a+1)^2 = a^4 + 2a^3 + 3a^2 + 2a + 1. Se D for um quadrado, então será da forma (a^2 + a + x)^2. Expandindo isso e comparando coeficientes, obtemos x = 1 ==> D = (a^2 + a + 1)^2. Como a^2 + a é par, raiz(D) = a^2 + a + 1 é ímpar. []s, Claudio. 2018-08-15 17:22 GMT-03:0

[obm-l] Re: [obm-l] Questão do Gandhi

2018-08-15 Por tôpico gilberto azevedo
Supondo que b>a, então b = a+1 Logo : D = a² + (a+1)² + (a*(a+1))² D = a² + a² + 2a + 1 + (a²+a)² D = 2a² + 2a + 1 + (a²+a)² D = 2(a²+a) + 1 + (a²+a)² D = (a²+a)² + 2(a²+a) + 1 (só organizei) Agora a sacada é perceber que está na forma x²+2xy+y² sendo x = a²+a e y = 1 Logo : D = (a²+a+1)² √D = a²+a

[obm-l] Questão do Gandhi

2018-08-15 Por tôpico Daniel Quevedo
Seja D = a^2 + b^2 + c^2, onde a e b são inteiros consecutivos e c = a•b. Então sobre a raiz quadrada de D podemos afirmar que: A) é sempre inteiro par B) algumas vezes é inteiro par, outras vezes não. C) algumas vezes é racional, outras vezes não. D) é sempre inteiro ímpar. E) é sempre irracional

[obm-l] Re: [obm-l] Re: [obm-l] Questão do IME

2018-07-14 Por tôpico Vanderlei Nemitz
Muito obrigado, Claudio! Bela solução! Em 13 de julho de 2018 13:35, Claudio Buffara escreveu: > Os prolongamentos de DM e EN se intersectam num mesmo ponto P pertencente > a AB. > Pra ver isso, repare que os triângulos DCM e PAM são semelhantes (razão de > semelhança = 2). > Idem para os triâng

Re: [obm-l] Re: [obm-l] Questão do IME

2018-07-14 Por tôpico wagner
Brilhante! Quoting Claudio Buffara : Os prolongamentos de DM e EN se intersectam num mesmo ponto P pertencente a AB. Pra ver isso, repare que os triângulos DCM e PAM são semelhantes (razão de semelhança = 2). Idem para os triângulos EFN e PNB. Como, no triângulo PDE (que é isósceles), vale PM/

Re: [obm-l] Questão do IME

2018-07-13 Por tôpico Ary Medino
Seguem algumas ideias para uma solução por vetores em R^3, mas precisa esboçar as figuras para ficar mais claro. - Sem perda de generalidade, assuma que os quadrados têm lado medindo 1. - Esboce a figura no R^3, sendo B a origem do sistema, o lado AB no eixo x e o lado BC no eixo y. - Chame de

[obm-l] Re: [obm-l] Questão do IME

2018-07-13 Por tôpico Claudio Buffara
Os prolongamentos de DM e EN se intersectam num mesmo ponto P pertencente a AB. Pra ver isso, repare que os triângulos DCM e PAM são semelhantes (razão de semelhança = 2). Idem para os triângulos EFN e PNB. Como, no triângulo PDE (que é isósceles), vale PM/PD = PN/PE = 1/3, concluímos que MN é para

[obm-l] Questão do IME

2018-07-13 Por tôpico Vanderlei Nemitz
Sejam dois quadrados ABCD e ABEF, tendo um lado comum AB, mas não situados num mesmo plano. Sejam M e N pertencentes, respectivamente, às diagonais AC e BF tais que AM/AC = BN/BF = 1/3. Mostre que MN é paralelo a DE. Alguém poderia ajudar? Obrigado, Vanderlei -- Esta mensagem foi verificada pelo

[obm-l] Questão da XXII olimpiada de mayo

2018-06-15 Por tôpico Pedro José
Bom dia! Alguém poderia postar uma solução de um problema da XXII olimpiada, mais especificamente o item b) :Dizemos que um número inteiro positivo é qua-divi se é divisível pela soma dos quadrados de seus dígitos, e além disso nenhum de seus dígitos é igual a zero. a) Encontre um número

[obm-l] Re: [obm-l] Questão sobre divisor primo

2018-06-06 Por tôpico Otávio Araújo
Só uma curiosidade: de onde é essa questão? Em qua, 6 de jun de 2018 11:38, Pedro Chaves escreveu: > Caros Colegas, > > Não consegui resolver a questão abaixo. Peço auxílio. > > Questão:Para cada inteiro positivo n, mostrar que todo divisor primo > de 12n^2 + 1 é da forma 6k +1, sendo k um i

[obm-l] Re: [obm-l] Questão sobre divisor primo

2018-06-06 Por tôpico Otávio Araújo
Só um detalhe que errei na digitação: 1 = (1/p) = (x.12/p) = ((-x).(-12)/p)= (-x/p)(-12/p) Em qua, 6 de jun de 2018 15:55, Otávio Araújo escreveu: > Tenho uma solução aqui: > Seja p um primo que divide 12n^2 +1, teremos que 12n^2 = -1 mód p. Seja x > o inverso multiplicativo de 12 módulo p em (Z

[obm-l] Re: [obm-l] Questão sobre divisor primo

2018-06-06 Por tôpico Otávio Araújo
Tenho uma solução aqui: Seja p um primo que divide 12n^2 +1, teremos que 12n^2 = -1 mód p. Seja x o inverso multiplicativo de 12 módulo p em (Zp)*, então n^2= -x mód p, portanto -x é resíduo quadrático módulo p. Denote (/) o simbolo de Legendre, teremos (-x/p)=1, mas 1=(1/p)=(x.12/p) (-x/p)(-12/p)

[obm-l] Re: [obm-l] Re: [obm-l] Questão sobre divisor primo

2018-06-06 Por tôpico Claudio Buffara
Na verdade é pra provar que se p é primo e divide 12n^2 + 1, então p é de forma 6k+1. 2018-06-06 12:50 GMT-03:00 Daniel Quevedo : > De uma maneira bem informal 6| 12n^2 , para qqr n inteiro. Logo 12n^2+1= 1 > (mod 6) ou seja é da forma 6k +1. > > Uma demonstração formal seria por indução finita

[obm-l] Re: [obm-l] Questão sobre divisor primo

2018-06-06 Por tôpico Daniel Quevedo
De uma maneira bem informal 6| 12n^2 , para qqr n inteiro. Logo 12n^2+1= 1 (mod 6) ou seja é da forma 6k +1. Uma demonstração formal seria por indução finita, onde P(0)= 12+1 = 13 = 1(mod 6) Se P(n) é verdade, logo P (n +1) = 12n^2 + 24n +12 + 1 = 6(2n^2 + 4n + 3) + 13 = 1 (mod 6) é vdd Acho q é i

[obm-l] Questão sobre divisor primo

2018-06-06 Por tôpico Pedro Chaves
Caros Colegas, Não consegui resolver a questão abaixo. Peço auxílio. Questão:Para cada inteiro positivo n, mostrar que todo divisor primo de 12n^2 + 1 é da forma 6k +1, sendo k um inteiro positivo. [https://ipmcdn.avast.com/images/icons/icon-envelope-tick-round-orange-animated-no-repeat-v

[obm-l] Re: [obm-l] Re: [obm-l] Re: [obm-l] Re: [obm-l] Questão de derivada

2018-04-23 Por tôpico Igor Caetano Diniz
na verdade eu não fiz rsrs. Eu queria ver um modo claro de mostrar. Se não puder usar L'Hospital, acho que tem que fazer uma sequência por baixo e uma por cima aplicando TVM em cada intervalo. Aí usa o fato dessa sequencia ser limitada, e monotona, portanto, convergente. Logo lim f'(xn) = L tanto

[obm-l] Re: [obm-l] Re: [obm-l] Re: [obm-l] Questão de derivada

2018-04-23 Por tôpico Artur Steiner
Eu li errado, temos que lim x --> 0 f' (x) = L. Assim, a Regra de l' Hopital conforme mostrei demonstra que, de fato, f'(c) = L. Mas o que vc fez não mostra que f'(c) = L. Artur Costa Steiner Em Seg, 23 de abr de 2018 14:31, Igor Caetano Diniz escreveu: > Se a questão tivesse um intervalo exp

[obm-l] Re: [obm-l] Re: [obm-l] Questão de derivada

2018-04-23 Por tôpico Igor Caetano Diniz
Se a questão tivesse um intervalo explícito [a,b] e diferenciável em todo ponto (a,b) exceto possivelmente num ponto c em (a,b) tal que lim f '(x) = L, x-> c, o que eu fiz estaria correto? 2018-04-23 14:11 GMT-03:00 Artur Steiner : > Como f é contínua em 0, então, pela regra de L'Hopital, > > lim

[obm-l] Re: [obm-l] Questão de derivada

2018-04-23 Por tôpico Artur Steiner
Como f é contínua em 0, então, pela regra de L'Hopital, lim x --> 0+ (f(x) - f0))/(x - 0) = lim x --> 0+ f'(x) = L Pela definição de derivada lateral, o limite do primeiro membro é a derivada à direita de 0. É só o que podemos concluir do enunciado. Nada garante que a derivada à esquerda de 0 se

[obm-l] Re: [obm-l] Re: [obm-l] Questão de derivada

2018-04-23 Por tôpico Igor Caetano Diniz
Então, Se existem os limites laterais, lim f ' (0-) = lim f ' (0+) então, defina q(x) = [f(x) - f(0)]/x. Para todo x<0, existe y1 entre x e 0 tal que f ' (y) = q(x). Analogamente para x>0, existe z1 entre 0 e x tal que f ' (z) = q(x). Defina r(x,0) a distancia de x para 0 Então, seja yn = yn-1 + r

[obm-l] Re: [obm-l] Questão de derivada

2018-04-23 Por tôpico Bernardo Freitas Paulo da Costa
2018-04-22 22:36 GMT-03:00 Igor Caetano Diniz : > Boa noite, > Gostaria de uma ajuda numa questão. Primeiro saber se pensei corretamente na > maneira (1) e se é possível resolver como pensei também na maneira (2). > Aí vai: > Questão 5.3.8 do livro do Stephen Abbot, Understanding Analysis: > > Assu

[obm-l] Questão de derivada

2018-04-22 Por tôpico Igor Caetano Diniz
Boa noite, Gostaria de uma ajuda numa questão. Primeiro saber se pensei corretamente na maneira (1) e se é possível resolver como pensei também na maneira (2). Aí vai: Questão 5.3.8 do livro do Stephen Abbot, Understanding Analysis: Assuma que f é contínua em um intervalo que contém o zero e difer

[obm-l] Re: [obm-l] Re: [obm-l] Re: [obm-l] Re: [obm-l] Questão de Combinatória

2018-03-30 Por tôpico Anderson Torres
Em 29 de março de 2018 15:37, Igor Caetano Diniz escreveu: > Vou mostrar a sua e a minha e aí se ele não aprender com as duas, tento > fazer devagar em casos menores. hehe > > Abraços Cláudio e obrigado =) > > 2018-03-29 15:17 GMT-03:00 Claudio Buffara : >> >> Sim. Acho essa uma solução bem mais e

[obm-l] Re: [obm-l] Re: [obm-l] Re: [obm-l] Re: [obm-l] Questão de Combinatória

2018-03-29 Por tôpico Claudio Buffara
Outra sugestão: proponha o problema de contar de quantas maneiras é possível arrumar N dominós 1x2 numa caixa 2xN. Fibonacci também aparece neste aí. A diferença é que, no dos bits, B(N) = F(N+2) enquanto que, no dos dominós, D(N) = F(N+1) (F é definida da forma usual, com F(1) = F(2) = 1) Ou ent

[obm-l] Re: [obm-l] Re: [obm-l] Questão de Combinatória

2018-03-29 Por tôpico Claudio Buffara
Sugestão de natureza didática: eu mostraria uma solução mais braçal, tal como a minha, e depois mostraria a solução recursiva. Moral: em geral vale a pena pensar no problema antes de sair escrevendo... 2018-03-29 15:17 GMT-03:00 Claudio Buffara : > Sim. Acho essa uma solução bem mais elegante. >

[obm-l] Re: [obm-l] Re: [obm-l] Re: [obm-l] Questão de Combinatória

2018-03-29 Por tôpico Igor Caetano Diniz
Vou mostrar a sua e a minha e aí se ele não aprender com as duas, tento fazer devagar em casos menores. hehe Abraços Cláudio e obrigado =) 2018-03-29 15:17 GMT-03:00 Claudio Buffara : > Sim. Acho essa uma solução bem mais elegante. > Mas também é mais sofisticada, e você falou que o aluno é prin

[obm-l] Re: [obm-l] Re: [obm-l] Questão de Combinatória

2018-03-29 Por tôpico Claudio Buffara
Sim. Acho essa uma solução bem mais elegante. Mas também é mais sofisticada, e você falou que o aluno é principiante. De todo jeito, acho que raciocinar recursivamente é uma habilidade que todo estudante de matemática deveria desenvolver. []s, Claudio. 2018-03-29 14:45 GMT-03:00 Igor Caetano Di

[obm-l] Re: [obm-l] Questão de Combinatória

2018-03-29 Por tôpico Igor Caetano Diniz
Olá Claudio Pensei numa solução agora que acredito que eu possa explicar e a pessoa irá entender: Para 1 bit, 2 possibilidades Para 2 bits, 3 Para 3 bits, basta separar em casos: Se for 0 _ _, cai no caso anterior. Se for 1 _ _ tem que ser 1 0 _ e, então, cai no caso anterior-1. Para 4 bits, sepa

Re: [obm-l] Questão de Combinatória

2018-03-29 Por tôpico Claudio Buffara
Sugestão: separe em casos em função do número N de 1’s na sequência. N = 0: 1 sequência N = 1: 8 sequências N = 2: 8*7/2 - 7 = 21 (No de sequências sem restrições menos o no de sequências com os dois 1’s adjacentes) N = 4: 2 N > 4: 0 O caso N = 3 é o mais chatinho pois tem mais subcasos, mas n

[obm-l] Questão de Combinatória

2018-03-29 Por tôpico Igor Caetano Diniz
Olá pessoal, Estou com uma questão de Combinatória e gostaria de uma solução didática para ela pq como eu fiz ficou complexo para um aluno que iniciou combinatória agora. segue a questão: Quantas sequências de 8 bits(com 0's e 1's) não têm dois 1 consecutivos? Como foi resolvida: usando variávei

[obm-l] Re: [obm-l] Re: [obm-l] Re: [obm-l] Re: [obm-l] Re: [obm-l] Questão de Cardinalidade

2018-01-21 Por tôpico Anderson Torres
Em 16 de janeiro de 2018 13:50, Bernardo Freitas Paulo da Costa escreveu: > 2018-01-16 1:10 GMT-02:00 Anderson Torres : >> Eu na verdade pensei ao contrário: >> >> Começamos com o conjunto de todos os subconjuntos de N. Cada conjunto >> será representado por uma string infinita de zeros e unzes, d

[obm-l] Re: [obm-l] Re: [obm-l] Re: [obm-l] Re: [obm-l] Re: [obm-l] Re: [obm-l] Questão de Cardinalidade

2018-01-16 Por tôpico Bernardo Freitas Paulo da Costa
2018-01-16 14:11 GMT-02:00 Igor Caetano Diniz : > Fala Bernardo, tudo certo? > Mas sera que eu conseguiria provar que esses números não seriam uma > quantidade enumeravel de pontos entre 0 e 1 e, então, como é enumeravel, eu > consigo pegar uma quantidade enumeravel em P(N) para esses pontos. Sim,

[obm-l] Re: [obm-l] Re: [obm-l] Re: [obm-l] Re: [obm-l] Re: [obm-l] Questão de Cardinalidade

2018-01-16 Por tôpico Igor Caetano Diniz
Fala Bernardo, tudo certo? Mas sera que eu conseguiria provar que esses números não seriam uma quantidade enumeravel de pontos entre 0 e 1 e, então, como é enumeravel, eu consigo pegar uma quantidade enumeravel em P(N) para esses pontos. Acha que seria ruim? Abraço On Jan 16, 2018 13:59, "Bernard

[obm-l] Re: [obm-l] Re: [obm-l] Re: [obm-l] Re: [obm-l] Questão de Cardinalidade

2018-01-16 Por tôpico Bernardo Freitas Paulo da Costa
2018-01-16 1:10 GMT-02:00 Anderson Torres : > Eu na verdade pensei ao contrário: > > Começamos com o conjunto de todos os subconjuntos de N. Cada conjunto > será representado por uma string infinita de zeros e unzes, da > seguinte forma: Se o conjunto contiver o natural x, o x-ésimo > caractere des

[obm-l] Re: [obm-l] Re: [obm-l] Re: [obm-l] Re: [obm-l] Questão de Cardinalidade

2018-01-16 Por tôpico Igor Caetano Diniz
Uma ideia legal Para provar que (-1,1) tem bijeção com R, seria usar f(x) = x/(x^2-1) provando que ela eh injetiva e sobrejetiva On Jan 16, 2018 01:20, "Anderson Torres" wrote: > Eu na verdade pensei ao contrário: > > Começamos com o conjunto de todos os subconjuntos de N. Cada conjunto > será r

[obm-l] Re: [obm-l] Re: [obm-l] Re: [obm-l] Questão de Cardinalidade

2018-01-15 Por tôpico Anderson Torres
Eu na verdade pensei ao contrário: Começamos com o conjunto de todos os subconjuntos de N. Cada conjunto será representado por uma string infinita de zeros e unzes, da seguinte forma: Se o conjunto contiver o natural x, o x-ésimo caractere desta string será 1; caso contrário, será 0. Botando zero

[obm-l] Re: [obm-l] Re: [obm-l] Questão de Cardinalidade

2018-01-15 Por tôpico Igor Caetano Diniz
Olá Sávio, Muito obrigado. Tava pensando em algo parecido mas agora voce esclareceu bastante. Abraços On Jan 15, 2018 16:55, "Sávio Ribas" wrote: > Boa tarde! > A primeira parte servirá para mostrar que a cardinalidade de IR é igual à > cardinalidade de [0,1]. > Não é difícil mostrar que a reta

[obm-l] Re: [obm-l] Questão de Cardinalidade

2018-01-15 Por tôpico Sávio Ribas
Boa tarde! A primeira parte servirá para mostrar que a cardinalidade de IR é igual à cardinalidade de [0,1]. Não é difícil mostrar que a reta tem a mesma cardinalidade que, por exemplo, o intervalo (-1,1) -- basta tomar a bijeção f: (-1,1) -> IR dada por f(x) = tg(pi*x/2). O passo seguinte seria mo

[obm-l] Questão de Cardinalidade

2018-01-15 Por tôpico Igor Caetano Diniz
Olá a todos, estou com uma dúvida para provar uma questão(Sem usar hipótese do contínuo) Prove que a cardinalidade do conjunto das partes dos números naturais é igual à cardinalidade dos reais, i.e., |P(N)| = |R| quem puder ajudar, agradeço. Abraços -- Esta mensagem foi verificada pelo sistem

[obm-l] Questão da OBM

2018-01-01 Por tôpico marcone augusto araújo borges
Seja a inteiro positivo e p um divisor primo de a^3 -3a +1 com p diferente de 3. Prove que p é da forma 9k + 1 ou 9k - 1, sendo k inteiro. Como resolver? -- Esta mensagem foi verificada pelo sistema de antivírus e acredita-se estar livre de perigo.

[obm-l] Re: [obm-l] Re: [obm-l] Questão

2017-08-28 Por tôpico Pedro José
Bom dia! Daniel, eu já me sinto gratificado quando consigo resolver algo. Não sou matemático, sou um pitaqueiro, com alto grau curiosidade e matemática é uma das minhas curiosidades preferidas. O que mais me fascina, é que sou totalmente crente em que um modelo matemático formulado com estrutura,

Re: [obm-l] Re: [obm-l] Questão

2017-08-25 Por tôpico Gabriel Tostes
Faltou so uma coisa, a ordem de 10 mod 23 é 11 nao 22. Entao o k= 2+11k > On Aug 25, 2017, at 12:28 AM, Daniel da Silva > wrote: > > Obrigado Pedro. > > Daniel Rocha da Silva > > Em 23 de ago de 2017, à s 19:31, Pedro José escreveu: > >> Boa noite! >> >> O difícil é achar o n. >>

Re: [obm-l] Re: [obm-l] Questão

2017-08-25 Por tôpico Gabriel Tostes
Confundi, eh 22 msm. :D > On Aug 25, 2017, at 12:28 AM, Daniel da Silva > wrote: > > Obrigado Pedro. > > Daniel Rocha da Silva > > Em 23 de ago de 2017, às 19:31, Pedro José escreveu: > >> Boa noite! >> >> O difícil é achar o n. >> >> Como o menor inteiro positivo que atende 10^a

Re: [obm-l] Re: [obm-l] Questão

2017-08-24 Por tôpico Daniel da Silva
Obrigado Pedro. Daniel Rocha da Silva > Em 23 de ago de 2017, às 19:31, Pedro José escreveu: > > Boa noite! > > O difícil é achar o n. > > Como o menor inteiro positivo que atende 10^a = 1 mod23 é a=22 > > E como 10^3 = 11 mod23. > > Temos que K + 1 = 3 +22*m com m natural > então k = 2

[obm-l] Re: [obm-l] Questão

2017-08-23 Por tôpico Pedro José
Boa noite! O difícil é achar o n. Como o menor inteiro positivo que atende 10^a = 1 mod23 é a=22 E como 10^3 = 11 mod23. Temos que K + 1 = 3 +22*m com m natural então k = 2 + 22*m. e n/2 = [10^(k+1) -11]/23 ==> n=2*[10^(k+1)-11]/23. Portanto as soluções serão (2+ 22*m; 2*[10^(3+22*m)-11]/23;

[obm-l] Questão

2017-08-23 Por tôpico Daniel da Silva
Boa tarde, Como saber quantos valores inteiros de N e K satisfazem a seguinte equação: 10^(K+1)=11+23N/2 Encontrei uma solução (N=86, K=2), mas como saber se é única? Obrigado, Daniel Rocha da Silva -- Esta mensagem foi verificada pelo sistema de antiv�rus e acredita-se estar livre de perigo.

[obm-l] Re: [obm-l]Re: [obm-l] Re: [obm-l] Re: [obm-l] Questão de teoria numérica

2017-08-01 Por tôpico Pedro Cardoso
Realmente. Se isso serve de desculpa eu escrevi isso assim que acordei. O que eu quis dizer é que não existem múltiplos de 2017 que terminem em 0 e que, ao serem divididos por 10, deixam de ser múltiplos de 2017. Para isso existir, 2017 teria que ter um número de fatores 2 diferente do número de

[obm-l] Re: [obm-l] Re: [obm-l] Re: [obm-l] Questão de teoria numérica

2017-08-01 Por tôpico Bernardo Freitas Paulo da Costa
Em 01/08/2017 08:14, "Pedro Cardoso" escreveu: > > Obrigado! Era exatamente isso que a questão anterior sugeria, usar o > princípio da casa dos pombos. > Uma coisa que percebi na sua dsmonstração é que o número encontrado > terminaria em 0s, mas como nenhum multiplo de 2017 também é multiplo de

[obm-l] Re: [obm-l] Re: [obm-l] Questão de teoria numérica

2017-08-01 Por tôpico Pedro Cardoso
Obrigado! Era exatamente isso que a questão anterior sugeria, usar o princípio da casa dos pombos. Uma coisa que percebi na sua dsmonstração é que o número encontrado terminaria em 0s, mas como nenhum multiplo de 2017 também é multiplo de 10 (2017 é primo) então também existe um multiplo de 2017 co

[obm-l] Re: [obm-l] Questão de teoria numérica

2017-07-31 Por tôpico Adilson Francisco da Silva
Salve! Construa uma sequência com 2018 números naturais da seguinte forma: 1 11 111 . . . 111...1 (2018 dígitos 1). Pelo princípio da casa dos pombos existe ao menos dois desses números que deixam o mesmo resto na divisão por 2017. Use o fato de que se dois números deixar o mesmo resto na d

[obm-l] Questão de teoria numérica

2017-07-31 Por tôpico Pedro Cardoso
Segue uma questão de teoria numérica da Olimpíada SESI de Matemática (AM): Mostre que existe um múltiplo de 2017 formado apenas pelos dígitos 0 e 1 (em base 10). Na olimpíada, a questão anterior sugere uma maneira de resolver, porém, estou interessado em outras demonstrações também. Se ninguém

[obm-l] Re: [obm-l] Questão de um vestibular do Acre

2016-12-23 Por tôpico Pacini Bores
Oi Wanderlei, Realmente, acredito que falte o ângulo theta, já que ele pede para usar sqrt(2)=1,4. Na verdade o comprimento da maca, para tocar os extremos nas paredes dos corredores, tem sua limitação dada por (p^(2/3)+q^(2/3)^(3/2) se imaginarmos a largura da maca desprezível. Abraços

[obm-l] Questão de um vestibular do Acre

2016-12-21 Por tôpico Vanderlei Nemitz
Boa tarde! Tentei resolver uma questão de um vestibular do Acre, mas parece que faltam informações, que talvez seja necessário supor. Como acho que não posso anexar um arquivo aqui, deixo um link que acessa a prova. É a questão *32*, de geometria plana. http://www.strixeducacao.com.br/vs-arquivos/

[obm-l] Re: [obm-l] Re: [obm-l] Questão de Física

2016-10-16 Por tôpico Luiz Antonio Rodrigues
Olá, Pacini! Muito obrigado! Um abraço! Luiz On Oct 16, 2016 10:38 AM, "Pacini Bores" wrote: > > > > Oi Luiz, > > o T para pequenas oscilações , T = 2.pi.sqrt(L/g) e com T´=5T= > 2.pi.sqrt(L/g´), onde g´= (P-q.E)/m. > > Logo teremos : (T^2).g = ((T´)^2).g´ ou seja g=25.g´ou g = 25(P-q.E)/m e > fa

[obm-l] Re: [obm-l] Questão de Física

2016-10-16 Por tôpico Pacini Bores
Oi Luiz, o T para pequenas oscilações , T = 2.pi.sqrt(L/g) e com T´=5T= 2.pi.sqrt(L/g´), onde g´= (P-q.E)/m. Logo teremos : (T^2).g = ((T´)^2).g´ ou seja g=25.g´ou g = 25(P-q.E)/m e fazendo as contas, encontramos E = 240N/C. Abraços Pacini Em 15/10/2016 13:49, Luiz Antonio Rodrigues

Re: [obm-l] Questão de Física

2016-10-15 Por tôpico regis barros
Olá LuizO segue fórum vc encontra no yahoo grupos.Física Básica é o nome do fórum. Regis Em Sábado, 15 de Outubro de 2016 14:24, Luiz Antonio Rodrigues escreveu: Olá, pessoal! Peço desculpas por postar uma questão de Física, mas preciso de ajuda... Já tentei resolvê-la muitas vezes, s

[obm-l] Questão de Física

2016-10-15 Por tôpico Luiz Antonio Rodrigues
Olá, pessoal! Peço desculpas por postar uma questão de Física, mas preciso de ajuda... Já tentei resolvê-la muitas vezes, sem sucesso. Não conheço um bom fórum de Física. Desde já agradeço qualquer ajuda. A questão é a seguinte: Um pequeno pêndulo simples é posto a oscilar entre duas superfície

[obm-l] Re: [obm-l] [obm-l] Questão Geometria

2016-10-10 Por tôpico Carlos Victor
Olá Vinicius, Seja R a intersecção de AO com BC. Seja T a intersecção da bissetriz de Será que alguém poria me ajudar na seguinte questão? > > * > > (Belarus) Seja O o centro do círculo ex-inscrito do triângulo ABC oposto ao > vértice A. Seja M o ponto médio de AC e seja P a intersec ̧ão

[obm-l] Re: [obm-l] Re: [obm-l] Re: [obm-l] [obm-l] Questão Geometria

2016-10-09 Por tôpico Jeferson Almir
Sei que o tópico não tem nada a ver com o problema proposto, mas já postei 2 problemas que não aparecem na caixa da lista e percebi que alguns receberam pois até responderam. Isso já aconteceu com alguém??? Em 9 de outubro de 2016 15:23, Israel Meireles Chrisostomo < israelmchrisost...@gmail.com>

[obm-l] Re: [obm-l] Re: [obm-l] Re: [obm-l] [obm-l] Questão Geometria

2016-10-09 Por tôpico Jeferson Almir
Perdão foi processado sim na Mail Archive acabo de constatar mas demorou alguns dias para aparecer. Valeu!! Em 9 de outubro de 2016 17:40, Jeferson Almir escreveu: > Sei que o tópico não tem nada a ver com o problema proposto, mas já postei > 2 problemas que não aparecem na caixa da lista e perc

[obm-l] Re: [obm-l] Re: [obm-l] [obm-l] Questão Geometria

2016-10-09 Por tôpico Israel Meireles Chrisostomo
Se vc não quiser receber mais emails da obm l envie um emeail para obm l Em 8 de outubro de 2016 13:15, Matheus Herculano < matheusherculan...@gmail.com> escreveu: > A resposta é para de me mandar isso > > Em 1 de out de 2016 20:00, "vinicius raimundo" > escreveu: > >> Será que alguém poria me a

[obm-l] Re: [obm-l] [obm-l] Questão Geometria

2016-10-08 Por tôpico Matheus Herculano
A resposta é para de me mandar isso Em 1 de out de 2016 20:00, "vinicius raimundo" escreveu: > Será que alguém poria me ajudar na seguinte questão? > > >1. > >(Belarus) Seja O o centro do círculo ex-inscrito do triângulo ABC oposto >ao vértice A. Seja M o ponto médio de AC e seja P a

[obm-l] Re: [obm-l] Questão Geometria

2016-10-05 Por tôpico vinicius raimundo
Obrigado Douglas Em quarta-feira, 5 de outubro de 2016, Douglas Oliveira de Lima < profdouglaso.del...@gmail.com> escreveu: > Bom vamos lá, não tem nada de bonito nessa resolução. > > Seja O o centro do ex-incirculo de ABC tangente ao lado BC, temos que AO é > bissetriz do ângulo BAC, seja Q a in

[obm-l] Re: [obm-l] [obm-l] Questão Geometria

2016-10-04 Por tôpico Douglas Oliveira de Lima
Bom vamos lá, não tem nada de bonito nessa resolução. Seja O o centro do ex-incirculo de ABC tangente ao lado BC, temos que AO é bissetriz do ângulo BAC, seja Q a intercessão de AO com BC, e J o pé da perpendicular tirada de O ao lado AC, sendo BAQ=x, nós teremos CAQ=ACB=x, AQB=OQC=2x. E OC é biss

[obm-l] [obm-l] Questão Geometria

2016-10-01 Por tôpico vinicius raimundo
Será que alguém poria me ajudar na seguinte questão? 1. (Belarus) Seja O o centro do círculo ex-inscrito do triângulo ABC oposto ao vértice A. Seja M o ponto médio de AC e seja P a intersec ̧ão das retas MO e BC. Prove que se ∠BAC = 2∠ACB, então AB = BP. -- Esta mensagem foi verif

[obm-l] Re: [obm-l] Re: [obm-l] Questão de vetores

2016-09-21 Por tôpico Bruno Lira
Tome N um ponto tal que MN seja paralelo a AB. Note que o triângulo ABC é semelhante ao triângulo NMC. Dá semelhança de triângulo temos que: NM/AB = MC/BC => NM/BC = 5/8 => NM =5AB/8. e NC/AC = MC/BC => NC/AC = 5/8 => NC = 5AC/8. AN = AC -NC = 3AC/8. Daí: vetor(AM) = vetor(AN) + vetor(NM)

[obm-l] Re: [obm-l] Questão de vetores

2016-09-19 Por tôpico Anderson Torres
Em 21 de agosto de 2016 00:59, Henrique N. Lengler escreveu: > Olá, > > Estou estudando vetores pelo livro "Vetores e uma iniciação à Geometria > Analítica" de Dorival A. de Mello e Renate Watanabe. > > Encontrei uma questão simples, mas que me deixou de cabelo em pé. Eu consegui > resolver, mas d

[obm-l] Questão de vetores

2016-08-20 Por tôpico Henrique N. Lengler
Olá, Estou estudando vetores pelo livro "Vetores e uma iniciação à Geometria Analítica" de Dorival A. de Mello e Renate Watanabe. Encontrei uma questão simples, mas que me deixou de cabelo em pé. Eu consegui resolver, mas de uma maneira meio feia. A questão é: *.Obs: Todas as duplas de letras sã

[obm-l] Re: [obm-l] Questão de teoria dos números

2016-03-27 Por tôpico Anderson Torres
Em 27 de março de 2016 19:20, Pedro Chaves escreveu: > Caros Colegas, > > Peço-lhes ajuda na questão abaixo. > > Sendo x um número inteiro qualquer e y um inteiro positivo, mostrar que > existe um inteiro k tal que: > --- ky é menor ou igual a x e (k+1)y é maior do que x. > k <= x/y <=

[obm-l] Questão de teoria dos números

2016-03-27 Por tôpico Pedro Chaves
Caros Colegas, Peço-lhes ajuda na questão abaixo. Sendo x um número inteiro qualquer e y um inteiro positivo, mostrar que existe um inteiro k tal que: --- ky é menor ou igual a x e (k+1)y é maior do que x. Abraços do Pedro Chaves __ --

RE: [obm-l] Questão interessante

2015-11-03 Por tôpico Esdras Muniz
Se não há um dígito que aparece 3 vezes, então cada digito 0, 1, ..., 9 aparece duas vezes. Então a soma dos dígitos de p^n é 90, então 9|p^n. -Mensagem Original- De: "marcone augusto araújo borges" Enviada em: ‎03/‎11/‎2015 07:32 Para: "obm-l@mat.puc-rio.br" Assu

[obm-l] Questão interessante

2015-11-03 Por tôpico marcone augusto araújo borges
Seja p um número primo, p > 3.Sabe-se que para um certo inteiro positivo no número p^n possui 20 dígitos, quando escrito na base 10.Prove que dentreesses dígitos existem pelo menos três iguais. Eu tenho a solução.Estou compartilhando. -- Esta mensagem f

  1   2   3   4   5   6   7   8   9   >